by giladedelman Wed Nov 10, 2010 2:29 pm
Thanks for the post.
Be careful! We don't need to assume anything here. We're being asked to describe Chang's criticism of Werth's argument. What does Chang think Werth is wrong about?
Werth concludes that if it's true that there's no "manic-depressive gene," it must be false that anyone has a genetic predisposition to manic depression. Clearly, his assumption is that a manic-depressive gene is the only thing that could cause this type of genetic predisposition.
Chang identifies this assumption indirectly and weakens Werth's argument by introducing a specific alternative to his assumption: interactions among genes, rather than one single gene, cause manic depression.
So Chang's issue with Werth is that he does presuppose only one possibility (that a single gene causes manic depression) when according to her, at least one other possibility (interactions among multiple genes cause it). So (A) is correct.
(B) is the opposite of what Chang says; she agrees with the evidence Werth cites.
(C) is incorrect because Chang doesn't address expertise, and Werth pretty clearly cites the right experts.
(D) is just way out. Werth doesn't disallow any kind of evidence.
(E) is incorrect because likelihood vs. possibility never comes up.
Does that answer your question?